Diskussion:Hydrostatisches Paradoxon

aus Wikipedia, der freien Enzyklopädie
Zur Navigation springen Zur Suche springen

Kraftvektoren[Quelltext bearbeiten]

Hallo. Ich habe einen Kommentar zu dem Bild mit den Kraftpfeilen abzugeben: Kraftpfeile haben immer einen Angriffspunkt, im oberen Teil des Bildes ist wohl als Angriffspunkt die Wasseroberfläche nten versetzt werden. Leider wird dadurch das Bild unübersichtlicher und muss neu gestaltet werden. --Quantenemitter 15:49, 24. Feb. 2009 (CET)Beantworten

Ich habe mir das Hydrostatische Paradoxon noch einmal durch den Kopf gehen lassen. Hiermit möchte ich mich für meine falschen Aussagen entschuldigen, denn diese hatten mit dem ruhenden Zustand von Flüssigkeiten nicht viel zu tun. Doch das ändert nichts an der Tatsache, dass ich dieses Paradoxon erklären möchte.

Ich habe nun den Artikel komplett umgestaltet, sodass (hoffentlich) wirklich nur mehr der ruhende Zustand gilt.

In der Formulierung des Hydrostatischen Paradoxon habe ich geschrieben, dass die Kraft und der Druck (meinen Überlegungen zufolge) auch unabhängig von der Bodenfläche sind. Gibt es Argumente dagegen?

Weiters interessiert mich euere Meinung zum Absatz mit der Beeinflussung des Messergebnisses bei der Messung:

Eigentlich kann man nicht feststellen, ob auf eine Stelle der Gefäßwand eine Kraft wirkt oder nicht. Die Kraft durch die Flüssigkeit wird immer zur schwächsten Stelle umgeleitet, an der die geringste Gegenkraft wirkt. Erst wenn man an diese Stelle ein Loch bohrt, weiß man ganz sicher, dass nun eine wirkt, doch damit hat man bewusst die Kräfte an diese Stelle der Hülle umgeleitet. Die Messung beeinflusst das Ergebnis!

Außerdem habe ich das oberste Bild ausgetauscht - es kann sein dass noch eine alte Version vom Cache angezeigt wird.

--Bd 22:15, 21. Okt 2004 (CEST)

Die Kraft kann man sehr wohl mit einem einfachen Manometer messen.
Ein Manometer mißt den DRUCK, nicht die KRAFT. Und die Kraft auf der Meßfläche des Manometers ist hier bliebig wenig hilfreich. gruss --Dreiundvierzig 10:43, 23. Dez 2004 (CET)
druck ist kraft pro fläche. daher ist selbstverständlich die kenntnis der größe der kraft auf die messfläche des manometers sehr hilfreich. die kraft auf die ganze gefäßwand dagegen nicht.
dieser ganze artikel ist ein beispiel dafür, warum unter Wikipedia:Was Wikipedia nicht ist Wikipedia dient nicht der Theoriefindung, sondern der Theoriedarstellung steht. statt auf eigene faust zu versuchen, dies zu verstehen, und die fortschrittte dieses verstehensprozesses in diesem artikel zu dokumentieren, sollte man sich ein paar vernünftige physikbücher hernehmen und sich an deren darstellung orientieren. (wo ist der abschnitt "literatur?) grüße, Hoch auf einem Baum 15:49, 13. Feb 2005 (CET)
Da gebe ich Dir 100%ig recht. So schön es auch zu sehen ist, wenn sich jemand aktiv mit einem Thema auseinandersetzt und dabei lernt - den Wiki-Leser bringt das nicht weiter.
Das Hydrostatische Paradoxon läßt sich leicht auf folgende Zusammenhänge reduzieren: "Gleiche Flüssigkeit - gleiche Füllhöhe - gleiche Bodenfläche - verschiedene Gefäßform --> gleiche Bodendruckkräfte" (siehe hierzu auch Bohl/Elmendorf: Technische Strömungslehre, 13. Aufl., S. 57).
Gruß! netzmeister 21:17, 25. Sep 2005 (CET)
Deine einfache Zusammenfassung birgt leider ein Missverständnispotential in sich. Die "gleiche Bodenfläche" ist nur nötig zu erwähnen, wenn die Kraft betrachtet werden muss (z.B. die Bodenfläche ist ein Hubkolben). Einfacher und allgemeiner ist es, nur den Druck zu betrachten. Dann ist die Größe der Bodenfläche egal und die Form im übrigen auch. Sobald man Gefäße betrachtet, die keine eindeutige Bodenfläche haben (z.B. Glas-Rundkolben in der Chemie), wird die Anwendung deiner einfachen Zusammenfassung schwierig.
Und du sprichst von "...druckkraft". Diejenigen, die in Physik nicht ganz sattelfest sind, missverstehen dies leicht als "Druck" und glauben dann, der Druck hängt von der Bodenfläche ab. Somit habe ich hier gewisse didaktische Bedenken. gruss --Dreiundvierzig 10:38, 26. Sep 2005 (CEST)
Ad 1: Da ich diese Kette auf die Bodendruckkraft beziehe, ist die Bodenfläche ein notwendiger Teil des ganzen. Beim Rundkolben habe ich ja auch eine (idealerweise infinitesimal kleine) Bodenfläche im Talpunkt.
Ad 2: Ich weiß, was Du meinst. Allerdings denke ich, daß ein solcher Artikel in korrekter Terminologie abgefaßt sein sollte - und da ist Druckkraft nunmal Druckkraft. Ich hätte also eher Bedenken, das ganze zu "verwässern". Gruß! netzmeister 23:00, 29. Sep 2005 (CET)

Artikel überarbeitet[Quelltext bearbeiten]

Ich habe den Artikel überarbeitet und nun alle Fälle aufgelistet. Ich bitte um die neuerliche Durchsicht bzw. Kontrolle auf Richtigkeit.

--Bd 23:05, 19. Okt 2004 (CEST)

Ganz kurz überflogen, sieht der neue Anfang gut ist. Aber das "Würfelmodell" unten müsste raus. --Pjacobi 00:06, 20. Okt 2004 (CEST)
Stimmt, das andere Verhalten der Würfel ist etwas verwirrend. Ich lasse die Würfel noch im Artikel, bis ich die bessere Erklärung fertig gestellt habe. --Bd 13:05, 20. Okt 2004 (CEST)

Zum Nachdenken[Quelltext bearbeiten]

Hallo,

erstmal danke für die Kritik. Bitte korrigiert mich, falls ich in meinen Vermutungen falsch liege. Doch ich bitte euch, folgendes durchzudenken:

In den Zeichnungen haben ich die Gefäße bewusst ohne festen Boden ausgestattet. Würde man blitzschnell den Boden des Gefäßes entfernen, so wirkt eine Kraft nach unten. Dieser Kraft wirkt eine andere Kraft, die von unten nach oben wirkt, entgegen. Den Begriff "Schwerekräfte der beteiligten Moleküle" habe ich bereits im Artikel erwähnt und werde ihn hier in der Diskussion nicht noch einmal erklären.

Unterscheiden wir einmal 4 Fälle bei einem Gefäß, das nach oben hin enger wird:

  • 1. Fall: Die Kraft von unten nach oben (hier als Gegenkraft bezeichnet) ist größer als die Kraft, die das Wasser und der Luftdruck eines Zylinders mit der selben Bodenfläche verursachen würde.
Ergebnis: Die resultierende Kraft drückt die Flüssigkeit nach oben.
  • 2. Fall: Die Gegenkraft ist größer als die "Schwerekräfte der beteiligten Moleküle" plus die zusätzliche Kraft durch den Luftdruck von oben, jedoch kleiner als die Kraft von unten im 1. Fall.
Ergebnis: Die Kräftedifferenz wirkt nach oben und belastet die Wand zusätzlich. Die Kraft von oben auf die Bodenfläche wird unabhängig von der Gefäßform. Der Druck hängt nur noch vom Füllstand und der Bodenfläche (--Bd 13:05, 20. Okt 2004 (CEST)) ab.
  • 3. Fall: Die Gegenkraft ist gleich der "Schwerekräfte der beteiligten Moleküle" plus die zusätzliche Kraft durch den Luftdruck von oben.
Ergebnis: Die Wand wird nach oben nicht zusätzlich belastet, seitlich natürlich schon. Die Kraft von oben auf die Bodenfläche entspricht der "Schwerekräfte der beteiligten Moleküle" plus die zusätzliche Kraft durch den Luftdruck von oben. Es fehlt im Vergleich zum 2. Fall die Gegenkraft der Wand nach unten - die Kraft von oben auf die Bodenfläche ist kleiner als beim 2. Fall. Eine kleinere Kraft auf die selbe Fläche bedeutet auch einen kleineren Druck. Die Flüssigkeit verbleibt im Gefäß. Misst man jedoch die Kraft auf eine Teilfläche des Bodens, die einen Querschnitt kleiner oder gleich dem geringsten Querschnitt des Gefäßes hat, so ist dieser Druck höher als jener auf die gesamte Bodenfläche.
  • 4. Fall: Die Gegenkraft ist kleiner als die "Schwerekräfte der beteiligten Moleküle" plus die zusätzliche Kraft durch den Luftdruck von oben.
Ergebnis: Die resultierende Kraft bewegt die Flüssigkeit nach unten.

Das allgemein bekannte Gesetz, wonach der Druck unabhängig von der Gefäßform ist, trifft genau auf den 2. Fall zu. Bei meiner Erklärung habe ich mich blindlings auf den 3. Fall eingeschossen und leider verabsäumt, den 2. Fall miteinzubeziehen.

Hallo Bd! Zu Fall 2: Der Druck hängt nur vom Füllstand ab, nicht von der Bodenfläche! Oder hattest Du die Kraft gemeint und "Druck" geschrieben? --Dreiundvierzig 09:02, 20. Okt 2004 (CEST)
Du hast Recht, ich meinte, die Kraft hängt von Füllstand und Bodenfläche ab, der Druck nur vom Füllstand. --Bd 13:05, 20. Okt 2004 (CEST)

--Bd 18:53, 19. Okt 2004 (CEST)

Ich mag die Rolle des Nörglers nicht, aber ein paar Dinge sehe ich hier, die noch grundsätzlich verkehrt sind:

Du unterscheidest Fälle mit "Die eine Kraft ist größer als die andere". Das h.P. betrifft einen unbewegten Fall und dort ist die Summe aller Kräfte, die auf eine Objekt (Wassermolekül) wirken, immer Null.

Der 1. und der 4. Fall sind natürlich nicht statisch. Bei den anderen Fällen gibt es trotz unterschiedlicher Kräfte von unten auf den Boden keine Bewegung. --Bd 18:29, 20. Okt 2004 (CEST)

An der Gefäßwandung wirken die Kräfte senkrecht zur Wandfläche nicht schräg.

In welchem Winkel die Kräfte genau wirken, ist nicht relevant, solange Kraft und die entgegengesetzte Widerstandskraft die gleiche Richtung besitzen. Eine Schräge kann man sich auch als eine Ansammlung winzig kleiner, rechteckiger Stufen vorstellen. Um die Richtung der Kraft genau zu erfahren, müsste man die atomare Struktur der Oberfläche berücksichtigen. --Bd 18:29, 20. Okt 2004 (CEST)

Die Sache mit dem offenen Gefäßboden finde ich sehr irritierend. Beim h.P. wird nur der Fall des vorhandenen Gefäßbodens betrachtet. Wenn der Boden blitzschnell weggenommen wird, dann kannst Du natürlich den Moment betrachten gerade bevor das Wasser anfängt zu fließen, aber da gelten andere Drücke und Kräfte als beim h.P. . In dem Sinne sind mir auch die braunen Kraft- oder Druckpfeile unklar. Entweder ist der Gefäßboden vorhanden, dann magenta, oder der Boden ist gerade geöffnet worden, dann Luftdruck (und nur so stark wie an den übrigen Stellen des Bildes auch).

Den Gefäßboden habe ich aus mehreren Gründen geöffnet. Zum Einen konnte ich die 4 Fälle nur dadurch vermitteln, dass ich andere Kräfte von unten auf die Bodenfläche wirken lasse. Zum Anderen kann der Druck im 3. Fall, der übrigens auch den Zustand beim verschlossenen Boden beschreibt, von der Gefäßform (Kegel) und der Größe der Fläche, auf die die Kraft wirkt, abhängen. Druck ist als Kraft pro Fläche definiert. Beim einem kegelförmigen Gefäß bedeutet eine Verzehnfachung der Fläche nicht unbedingt eine Verzehnfachung der Kraft auf diese größere Fläche. Das Messergebnis hängt also davon ab, wie groß die Fläche ist, auf die die Kraft wirkt. Ist die Messfläche extrem klein, so misst man natürlich immer den selben Druck, unabhängig von der Gefäßform. Die Gefahr ist jedoch, dass man in die Formel Kraft = Druck*Fläche diesen selben Druck und die gesamte Fläche des Bodens einsetzt und auf ein eigentlich viel zu hohes Ergebnis kommt (Als Annahme gilt der 3. Fall in einem kegelförmigen Gefäß.). --Bd 18:29, 20. Okt 2004 (CEST)

Der Luftdruck wird normalerweise nur dort betrachtet, wo er an der Gefäßöffnung auf die Flüssigkeit wirkt. Das Gefäß wird als starr angenommen und damit spielt der Luftdruck auf die Gefäßwandung keine Rolle. Wir haben hier ja keinen wassergefüllten Luftballon.

Stimmt. Ich hätte den Luftdruck auf die Gefäßwand einfach weglassen können und stattdessen die Gegenkraft der Gefäßwand erhöhen können. Meiner Meinung nach ist es physikalisch eleganter, diesen doch miteinzubeziehen. --Bd 18:29, 20. Okt 2004 (CEST)

Beim obersten Bild des Artikels sind die Druckpfeile, der Text unterm Bild und der Text rechts daneben noch falsch. --Dreiundvierzig 13:22, 20. Okt 2004 (CEST)

Warum? Es ist die Kraft eingezeichnet (schwarze Pfeile) und die Fläche, auf die sie wirkt (grün). Aus Beidem berechnet sich bekanntlich der Druck. Ich habe diese Darstellung deshalb gewählt, damit man gleich sieht, wo der Druck herrscht. --Bd 18:29, 20. Okt 2004 (CEST)

Erklärung ?[Quelltext bearbeiten]

Hier fehlt mir eine Erklärung. Warum ist das so? Jpp 20:59, 26. Jul 2004 (CEST)

Physikalische Realität[Quelltext bearbeiten]

Hallo Bd!

Du hast einen gut strukturierten und wunderbar bebilderten Artikel geschrieben, der sich auch gut lesen lässt. Und jetzt kommt der Wermutstropfen (wie sag ich 's einem engagierten Wikipedianer?): Die Beschreibung des Sachverhalts ist leider nicht so, wie in der physikalischen Realität :-(

Ursache deines Missverständnisses ist wohl der Text auf den Seiten der FH-Merseburg, der die Sache nämlich auch nicht trifft.

Und jetzt der Versuch, die Hydrostatik in Kürze zu beschreiben. Wenn's nicht verständlich gerät, frag einfach zurück:

  • In einer waagerechten Ebene einer Flüssigkeit ist der Druck überall gleich! Du hast an einigen Stellen geschrieben, dass auf dem Gefäßboden unterschiedliche Drücke herrschen. Nein, er ist überall gleich.
Bei einem verschlossenen Boden gebe ich dir vollkommend Recht. Bei einer Öffnung im Gefäßboden hängt zum Beispiel beim kegelförmigen Gefäß der Druck, von der Größe der Öffnung und von der Kraft, die von unten wirkt, ab. Die Kraft von oben auf den Boden verteilt sich, sodass diese bei schwächeren Stellen (schwache Stelle = Stelle, an der geringere Gegenkräfte wirken) mehr zur Geltung kommt.
--Bd 18:53, 19. Okt 2004 (CEST)
    • Würde der Druck in der Waagerechten unterschiedlich sein, würde solange Flüssigkeit fließen, bis gleiche Druckverhältnisse hergestellt sind. D.h. die Klötzchenstabel in Deinen Bildern üben entgegen Deiner Annahme eben doch seitliche Kräfte aufeinander aus.
    • In einem ruhenden System (hier: die unbewegte Flüssigkeit im Gefäss) ist die Summe aller Kräfte, die auf ein kleines Flüssigkeits"teilchen" wirken, Null (wäre sie nicht Null, würde sofort Bewegung entstehen). Druckunterschiede in einer Flüssigkeit kann es nur geben, wenn zusätzlich zum Druck noch weitere Kräfte wirken. Dies ist hier die Gewichtskraft. Und die wirkt nur senkrecht. Also können auch nur in senkrechter Richtung Druckunterschiede auftreten.
  • Und wie erklärt sich nun das Paradoxon?
    • Beim trichterförmigen Gefäß wird die Gewichtskraft von den schrägen Seitenwänden aufgenommen. Soweit ist Deine Aussage richtig. Die Richtung der Kraft ist jedoch senkrecht zur Behälterwand und nicht nach unten.
    • Aber wo kommt die zusätzliche Kraft bei den Gefäßen her, die nach oben eine kleinere Öffnungsfläche haben als der Boden? Es ist die Gefäßwandung, die eine Kraft auf die Flüssigkeit ausübt, denn auf den Druck der Flüssigkeit reagiert die Behälterwand mit Gegendruck. That's it.
Meine Vermutung/Theorie: Die zusätzliche Kraft durch den Gegendruck der Behälterwand entsteht erst dann, wenn die Kraft, welche von unten auf die gesamte Bodenöffnung wirkt, größer als die Gewichtskraft der "beteiligten Moleküle" plus der Kraft durch den Luftdruck wird. Je größer die Kraft wird, die von unten wirkt, desto mehr wird die Wand nach oben hin belastet. Es muss daher eine bestimmte Kraft von unten geben, ab der die Kraft nach oben auf die Wand entsteht.
--Bd 18:53, 19. Okt 2004 (CEST)
  • In einigen Bildern hast du den Druck P mit einem Richtungspfeil gemalt. Also innerhalb der Flüssigkeit ist an einem Ort der Druck von allen Seiten gleich gross. Dort, wo der Flüssigkeitsdruck auf die Behälterwand wirkt, gibt es dann natürlich eine Kraft F mit Richtung.

Ich hoffe, ich habe Dir jetzt nicht zusehr den Tag versaut. gruss --Dreiundvierzig 17:25, 13. Okt 2004 (CEST)

Das Kernproblem[Quelltext bearbeiten]

Dieser Teil ist falsch: wobei diese Fläche nicht größer als die geringste horizontale Querschnittfläche in jeder Höhe des Gefäßes sein darf.. Diese Einschränkung gilt nicht. --Pjacobi 10:17, 14. Okt 2004 (CEST)

Und die ERklärung mit den Würfeln ist unglücklich: Würfeltürme verhalten sich eben eher wie Festkörper, und damit tritt das h.P. nicht auf. Das Paradoxon am j.P. ist ja gerade, dass die Kraft auf den Boden abnimmt wenn man die Flüssigkeit einfriert, wenn das Gefäß open enger als unten ist. --Pjacobi 10:32, 14. Okt 2004 (CEST)
Hallo Pjacobi! Das h.P. hat nix mit einfrieren zu tun. Wie kommst Du dadrauf? --Dreiundvierzig 11:28, 14. Okt 2004 (CEST)

Die Formulierung des h.P:, die ich kenne, und in der die Paradoxie gut zum Ausdruck kommt, lautet sinngemäß:

Eine Flüssigkeit in einem Gefäß (dass oben enger ist als unten) übt eine größere Kraft auf den Gefäßboden aus, als ein Festkörper gleicher Form und Dichte.

Pjacobi 11:37, 14. Okt 2004 (CEST)

Mir ist die etwas allgemeiner gehaltene Formulierung geläufig, dass die Kraft auf den Gefäßboden unabhängig von der Gewichtskraft der Flüssigkeit ist (sondern nur von der Flüssigkeitshöhe abhängt).
Nochmal zu Einfrieren: Wenn es sich bei der Flüssigkeit um Wasser handelt und das Gefäß kegelförmig ist, hast Du gute Chancen, dass Du nach dem Einfrieren die Scherben Deines Gefäßes aufsammeln musst ;-)
Nun schaun wir mal, was Bd dadraus macht für den Schreibwettbewerb --Dreiundvierzig 12:34, 14. Okt 2004 (CEST)

Die Formulierung ist allgemeiner (sollte aber noch die Dichte und g enthalten), stellt das Paradoxe aber nicht so schöne heraus. Beim Wasser klappt das Einfrieren aus erwähnten Gründen nicht, das ist schon klar. --Pjacobi 13:22, 14. Okt 2004 (CEST)


Danke an Pjacobi! "Eine Flüssigkeit in einem Gefäß (dass oben enger ist als unten) übt eine größere Kraft auf den Gefäßboden aus, als ein Festkörper gleicher Form und Dichte" ist genau der Satz, der das Paradoxon m.E. perfekt ausdrückt. Ich weiß jetzt zwar immer noch nicht warum das so ist, d.h. habe nach wie vor keine anschauliche Erklärung (ich muss es halt hinnehmen, dass der experimentelle Befund so ist), aber dass es halt genau der Unterschied zwischen *Flüssigkeit* und *Festkörper* ist, der das Paradoxon ausmacht, finde ich von zentraler Bedeutsamkeit. --84.59.229.45 18:01, 10. Aug. 2013 (CEST)Beantworten

Anderer Ansatz?[Quelltext bearbeiten]

M.E. sollte die Erklärung allgemein über den Druck erfolgen, und darauf aufbauen, dass in Hydrostatik kleine Stempel hohe Drücke erzeugen, am besten mit einem Vergleich über das Hebelgesetz. Daraus ergibt sich dann, dass trotz des geringeren Gewichts, die Schichten kleinen Querschnitts im gleichen Maße zum Gesamtdruck beitragen.

Was meint Ihr?

Pjacobi 18:50, 20. Okt 2004 (CEST)

Erklärung über Druck: ACK! Hebelgesetz: Eher nicht, da das auch wieder mit Kräften arbeitet. Der Schlüssel zum Verständnis des h.P. ist IMO die Stelle, wo eine dünne, waagerechte Wasserschicht an die schräge Gefässwand stößt. Dort wo keine Gefässwand ist, erfolgt die Druckerhöhung über die Gewichtskraft des Wassers aber an der Gefässwand muß der Gegendruck der Wand mit in die Betrachtung. Die Kraft der Gefässwand muß über Vektorzerlegung in eine senkrechte und eine waagerechte Komponente zerlegt werden. Die waagerechte Komponente hat als Gegenkraft diejenige von der gegenüberliegenden Gefässwand (exakt wäre natürlich das Ringintegral um die dünne Wasserscheibe = 0). Die senkrechte Komponente ist dann sozusagen der "Ersatz" für die fehlende Wassersäule an der Stelle. --Dreiundvierzig 11:55, 21. Okt 2004 (CEST)


Datei:Paradoxon.PNG

Hallo Kino! Die Skizze geht genau in die Richtung, die ich mir vorstelle. Die grauen Pfeile ausserhalb des Gefäßes würde ich aber weglassen. --Dreiundvierzig 11:24, 25. Okt 2004 (CEST)

Diskussion im Schreibwettbewerb[Quelltext bearbeiten]

von Benutzer:Bd - (nachgetragen -- Necrophorus 19:06, 17. Sep 2004 (CEST))

Schön bebilderter Artikel. Ein paar Anmerkungen:
Der erste Satz liefert keine Definition des Themas. Zum ganzen einführenden Abschnitt: Was genau ist denn jetzt das (wenn auch nur scheinbare) Paradoxon?
Abschnitt Theorie: Kürzerer und längerer Abschnitt sind nicht aufeinander abgestimmt. Zumindest erwähnt der kürzere nichts von Luftdruck und sagt auch nichts zu den evtl. kleineren Teilflächen aus.
Der Gebrauch der ersten Person, also auch des "wir" ist in Enzyklopädieartikeln eher unüblich.
Kommunizierende Röhren: Zumindest der bunte Text ist recht aufdringlich - vielleicht sollte man den durch Normaltext ersetzen.
Einordnung: Welche Bedeutung hat der Effekt in der Technik, wo kommt er in der Natur vor (Stichwort: Artesischer Brunnen).
Geschichte: Wann wurde das Paradoxon erstmals beschrieben und erklärt?
Literatur fehlt noch
Grüße --mmr 03:34, 21. Sep 2004 (CEST)

Erläuterung der Theorie[Quelltext bearbeiten]

Der Absatz Erläuterung der Theorie erscheint mir nicht 100% richtig. Der Rest ist soweit gut. Ich kenne mich damit aber leider nicht gut genug aus, daher könnten meine Zweifel auch falsch sein. MFG, GFJ 14:15, 12. Jan 2005 (CET) P.S.: Ich werde nicht mehr auf diese Diskussionseite schauen, also hier direkt für mich zu antworten ist sinnlos.

Anschaulichkeit[Quelltext bearbeiten]

Ich finde den Artikel etwas unanschaulich.

Mit hat folgende Überlegung weitergeholfen: Der zusätzliche Druck rührt tatsächlich von den Gefässwänden her. Man stelle sich einen auf einer Platte umgekehrt stehenden Trichter vor, in dem man oben Wasser hineingiesst. Der Trichter wird ab einem bestimmten Füllgrad vom Wasser angehoben werden. Das zeigt deutlich das das Wasser von unten gegen die Gefässwand drückt, wie auch die Gefässwand gegen das Wasser, und genau um diesen Druck vergrössert sich der der Wassersäule.

Damit hat man gleich das paradoxe am Problem gelöst. Mir war nämlich die Vorstellung zuwieder dass eine Flüssigkeitssäule mehr Druck ausüben kann als ihrem Eigengewicht entspricht. Kann sie selbst auch nicht, sie kann nur das umgebende Gefäss "erleichtern" und dieses zusätzliche Gewicht als Druck nach unten vermitteln.

Bilder[Quelltext bearbeiten]

Ich bin auf die alten Bilder gestoßen, falls sie noch jemand braucht. --Crux 21:17, 19. Mär 2006 (CET)

Und hier die neuen Bilder zum Thema:

--Ost38 15:49, 26. Mär. 2008 (CET)Beantworten

Im Prinzip doch kein Unterschied zur Hydraulik[Quelltext bearbeiten]

Die unterste Lage der Moleküle in einer dünnen Flüssigkeitssäule kann ja wohl nicht erkennen, woher die Last über ihr herrührt. Statt von dem Gewicht der Säule kann der Druck also ebenso von einem Kraftkolben eines hydraulischen Systems herführen, wie in einer Bremsanlage. Dann kann man das ganze auch waagerecht legen, sieht völlig anders aus, funktioniert aber: Die Kräfte auf die Kolbenflächen verhalten sich wie die Flächen zueinander. Trotzdem habe ich mit dem h.P. ein Problem, dass ich schon lange mit mir rumschleppe: Eine Staumauer muss eine bestimmte Dicke haben, die wesentlich von der Wasserhöhe abhängt. Nun stelle ich mir zwei Staumauern vor, die sich wenige mm gegenüber stehen. Laut hydraulischen Gesetzen müssten die Mauern genauso dick bleiben, obwohl sich nur ein paar Liter Wasser dazwischen befinden!!! Das kann ich einfach nicht glauben. Ich habe schon viele Physiklehrer u.a. vor das Problem gestellt - alle machten erstaunte Augen. Peter Buschmann, Bielefeld

Und mit einem exakt senkrecht aufgestellten Plastikstrohhalm ausreichender Länge lässt sich auch das dickste Fass oder auch der sicherste Safe sprengen. Das ist doch gerade das paradoxe an der Sache, oder? ;-) Grüße aus Bielefeld nach Bielefeld! --92.201.241.247 14:38, 26. Feb. 2011 (CET)Beantworten
@Peter Buschmann. Ist aber so. Am besten illustriert das das Bild mit der Kapillare über dem Faß, bzw. das historische Experiment von Pascal 1648. Es gibt nur einen Unterschied: Wären die Staumauern bei der (geringen) Wassermenge nicht dicht, ergösse sich nur kurzzeitig ein kleines Rinnsal, durchbrechend durch einen kleinen Riß, in die beiden anschließenden Täler. Eine wesentliche Ursache für den großen Aufwand bei Staumauern ist die große Fläche, auf die der Druck wirkt. Der Druck selbst ist gar nicht so groß: Die Mauer ist typisch um die 100 Meter hoch, das gibt am Staumauerfuß ein paar Atmosphären Druck. In meinem Fahrradreifen ist Druck für eine 50 Meter hohe Staumauer. Allerdings hat die Staumauer ein paar Quadratmeter groß (mal den Stausee_Mooserboden als Beispiel, Moosersperre: Laut Bild grob dreieckig, Grundlinie 0,5 km, Höhe 0,1 km macht Fläche . Der Druck ist nicht auf der ganzen Fläche minimal, für eine einfache Abschätzung gehe ich von einem mittleren Druck von 3 bar aus (Wassertiefe nur ca. 30 Meter, die Mauer wird ja nach unten schmaler, Maximaldruck mehr als dreimal so groß).
. Das entspricht der Gewichtskraft von 100.000 Tonnen auf der Erdoberfläche. Die tatsächliche Gewichtsstaumauer besteht aus 670.000 m^3 Beton, das reicht also wahrscheinlich.

Achja: Wer meinen mittleren Druck ersetzen will, darf gerne die genaue Form der Staumauer rausfinden und das ganze mal integrieren.

Zurück zu Deinem Beispiel: Ja, wenn man ein solches dreieckiges Tal nimmt, zwei Staumauern auf Millimeter genau glattpoliert, auf 1 cm zusammenstellt und dann 250 Liter Wasser in den Spalt reintrichtert (man kann ja mehrere Trichter gleichzeitig nehmen, dann dauerts nicht so lang), dann müssen die beiden Staumauern eine Kraft aufbauen, die einem Gewicht von ungefähr hunderttausend Tonnen entspricht. --Blauer elephant (Diskussion) 10:50, 11. Nov. 2013 (CET)Beantworten

Ein Abschnitt und ein Bild kann mE raus[Quelltext bearbeiten]

Hallo zusammen,

ich hab mal ein klein wenig gestrafft und präzisiert und das ganze auf den Druck statt auf die Gewichtskräfte umgestellt. Eins gefällt mir überhaupt nicht, ich scheue allerdings noch davor, einfach zu löschen.

  • Der Text des Abschnitts Erklärung des Effektes in Kommunizierenden Röhren ist m.E. redundant zum Abschnitt darüber. Ohne Diskussion möchte ich ihn nicht rauswerfen.
  • das Bild mit den vielen Pfeilen geht mir zu sehr von den Gewichtskräften und nicht vom Druck aus. Es passt nicht zum Thema.

Ich bitte die anderen Autoren um Vorschläge und Meinungen.

Unsignierter Beitrag von mir selber: [1] vom 28.11.2011, 21.48 Uhr --Blauer elephant (Diskussion) 21:04, 19. Aug. 2013 (CEST)Beantworten

Scheinbares Paradoxon[Quelltext bearbeiten]

Sollte man nicht im Grunde von einem "scheinbaren" Paradoxon sprechen (da ja keine echte Singularität oder dergleichen vorliegt)? ("Das Hydrostatische Paradoxon (...) ist das scheinbare Paradoxon (...) -- Cancun 07:40, 9. Mär. 2012 (CET)

Ich bin Physik-LKler und finde es ausgesprochen paradox, und das auch noch nach Jahren, in denen mir das Thema immer mal wieder über den Weg gelaufen ist. Also ich finde es überhaupt nicht "scheinbar". --84.59.229.45 18:01, 10. Aug. 2013 (CEST)Beantworten

Zum Bild mit den 5 "Behältern" bzw. dem 5-teiligen kommunzierenden "Röhren"system (File:Hydrostatisches Paradoxon3.svg)[Quelltext bearbeiten]

Soweit ich weiß, hat Druck keine Richtung, Druck ist halt da oder nicht. Erst auf einer Grenzfläche oder allgemeiner Fläche kann man von einer Art "Richtung" des Drucks sprechen (und zwar inkarniert in Form einer Kraft in jedem Punkt der Fläche in Richtung der Flächennormalen). Insofern verstehe ich bei dem Bild schon mal nicht, wieso die Kraftpfeile alle parallel sind: Sollten sie nicht orthogonal zu den Wänden der Gefäße sein?

Weiterer Punkt: Beispiel der "Weinkaraffe" (also ein kegelförmiger Behälter mit r_unten > r_oben). Dabei wird immer argumentiert, dass das Glas "von oben" einen Druck ausübt (im Bild die rosa Pfeile bei z.B. dem vulkanförmigen Gefäßteil), der sich dann auch auf die unteren Wasserschichten auswirkt und damit auf der Höhe der untersten Wasserschicht dafür sorgt, dass dort der Druck so hoch ist, wie er nach der Formel rho * g * h ist.

Das finde ich (mal ganz abgesehen davon, dass die Kausalität auf den Kopf gestellt wird: Das *Wasser* erzeugt doch zunächst den Druck, nicht das Glas, oder?) vollkommen unlogisch. Was wäre im (Fast-)Vakuum los, wenn wir das Gefäß ohne Inhalt betrachten? Sprich in genügend großem Abstand von der Erde, so dass die Atmosphäre praktisch nicht mehr vorhanden, aber noch die Massenanziehung da ist. Dort ist kein Medium, folglich kann das Glas überhaupt keinen Druck auf irgendwas auswirken, außer auf sich selbst. Wieso sollte es, füllt man jetzt Wasser ein [das natürlich sofort verdampfen würde im Weltall, schon klar... ignorieren wir das Problem mal], plötzlich damit anfangen? Das trichterförmige Gefäß im Bild kann einen *Gegen*druck nach oben aufbauen, *weil* es unter Belastung steht. Aber das trifft doch nicht auf die Karaffe zu (?). --84.59.229.45 18:44, 10. Aug. 2013 (CEST)Beantworten

Hallo,
ich kann Deiner Argumentation folgen und schließe mich an. Allerdings ist die "Erklärung" mit den unsäglichen Pfeilchen von den Gefäßwänden aus auch im Gerthsen drin. Eine Herleitung, die völlig ohne die Form des Gefäßes und daher auch ohne Gefäßwände auskommt, wurde mir rausgelöscht mit der Begründung, dass Herleitungen in Artikeln nichts verloren hätten, das habe ich auch eingesehen. Der Kern der Argumentation ist, dass das Erzeugen eines (infinitesimal kleinen) leeren Raumes, z.B. durch Aufziehen eines Kolbens gegen den Umgebungsdruck, das Wasser genau um die Tiefe bewegen muss. Daraus errechnet sich ganz zwanglos der hydrostatische Druck. Wenn man noch den Luftdruck draußen lassen will, kann man das Innere des Kolbens durch ein Luftrohr mit der Luft über dem Wasser verbinden und arbeitet dann nur noch gegen den Wasserdruck.
Das mit den Pfeilchen an den Gefäßen ist auch noch aus einem anderen Grund Müll: Befestigt man zwei kurze Rohre (gleiche Wandstärke) horizontal so, dass der untere Rand des oberen Rohres auf derselben Höhe liegt wie der obere des unteren Rohres (wegen der Endlichkeit des Rohrdurchmessers natürlich nebeneinander), und bohrt ins untere Rohr ein Loch von oben rein und ins obere ein gleich großes von oben, wird man, da der hydrostatische Druck für beide Löcher gleich ist, finden, dass die Austrittsgeschwindigkeit des Wassers an beiden Löchern identisch ist. Beim unteren wird das Wasser natürlich später langsamer, da es aufsteigen muss, beim oberen schneller, weil es direkt fällt. Liegt daran, dass es dem hydrostatischen Druck eben genau egal ist, in welche Richtung er wirkt.
Das Gefäß hat nur die Aufgabe, das Wasser drinnen zu halten, und damit es das kann, muß es an jedem Punkt seiner (inneren) Oberfläche dem hydrostatischen Druck standhalten, oder mit anderen Worten, jederzeit einen Gegendruck gewährleisten, oder nochmal mit anderen Worten, auf jedem infinitesimalen Flächenelement der Kraft, die das Wasser auf das Gefäß ausübt, eine Gegenkraft entgegensetzen. Sonst nichts. --Blauer elephant (Diskussion) 22:56, 17. Aug. 2013 (CEST)Beantworten
Bißchen späte Antwort: Der statische Druck in einem Fluid ist ein symmetrischer (und insofern richtungsloser) Tensor. Eine richtungsbehaftete Kraft entsteht erst, wenn dieser Drucktensor mit einem, natürlich gerichteten, Flächenvektor multipliziert wird, welche mathematische "Feinheit" im Schulunterricht gewöhnlich nonchalant geschlabbert wird, weil Tensoren in der Schule nicht vorkommen. - Wo kommt die Kraft her? Ein sich unten aufweitendes Gefäß mit Flüssigkeit übt auf die als eben und horizontal angenommene Bodenfläche ggf. eine größere Kraft aus, als dem Gewicht der Flüssigkeit entspricht - Kraft aus dem Nichts? (Und diese Kraft läßt sich auch direkt messen: man nehme ein solches Gefäß ohne Boden, dessen Rand man sehr dicht über einer Waagschale anordnet, so daß durch den Spalt praktisch keine Flüssigkeit austritt, und lese die Waage ab: sie zeigt als Kraft "Druck mal Fläche" an. Wie kann das sein, wenn die Flüssigkeit weniger wiegt als das angezeigte Gewicht?) Die Lösung ist: Die Flüssigkeit "borgt" sich Kraft aus der Gefäßwand: ihr Druck übt auf die nach innen geneigten Gefäßwandteile eine Auftriebskraft aus, die durch das Gewicht des Gefäßes oder eine Haltekraft kompensiert werden muß, denn sonst schwimmt das Gefäß auf und vergrößert den Spalt, wodurch die Flüssigkeit natürlich sofort auslaufen würde. Man muß also ggf. von oben auf das Gefäß drücken, damit es nicht aufsteigt, und diese äußere Andruckkraft addiert sich zum Eigengewicht der Flüssigkeit und bewirkt, daß sie die dem Druck entsprechende Kraft überhaupt auf die Bodenfläche ausüben kann. Wenn man gleichzeitig zwei Waagen verwendet, verschwindet das scheinbare Paradoxon sofort: Zunächst wird das Gefäß ohne Boden unten mit einer dünnen Platte oder Membran verschlossen, gefüllt und an einer Waage aufgehängt. Die zeigt dann das Gewicht aus Gefäß plus Flüssigkeit an. Nun wird sehr dicht unter dem Gefäß eine zweite Waage angeordnet, die erst einmal gar nichts anzeigt, weil sie durch die über ihr schwebende Last nicht belastet wird, denn die hängt an der oberen Waage. Wenn man nun den Boden entfernt - das kann z. B. eine Eisplatte sein, die schmilzt, oder es wird einfach ein dünnes Loch in die Platte hineingepiekt bzw. durch Herausziehen eines Stopfens geöffnet - dann wirkt der Druck anstatt auf den nun perforierten Boden auf die untere Waage, die deswegen ein entsprechendes "Gewicht" anzeigt. Um genau diese Gewichtskraft verringert sich aber die Anzeige der oberen Waage, u. U. sogar bis ins Negative, d. h. das Gefäß zieht nicht mehr an der Waage nach unten, sondern drückt nach oben (wofür Gefäß, Halterung und Waage ausgelegt sein müssen). Entscheidend ist: die Summe aus beiden Waageanzeigen bleibt dabei gleich - zusätzliches Gewicht unten resultiert in fehlendem oben, von wo die Flüssigkeit via Gefäßwand die erforderliche Kraft hernimmt. --77.0.8.110 19:00, 5. Mai 2022 (CEST)Beantworten
Diese Erklärbärerei ist mir zu verschwurbelt.
  • Der Versuch, sich ein Paradoxon mittels Alltagserfahrung vorzustellen, muss Aua in Kopf machen.
Eine Ansage mit „Tensor“ ist da schon sehr viel zielführender.
  • In die Berechnung geht schlicht und einfach nicht ein, was für Querschnitte oberhalb unterwegs sind.
  • Wenn schon, dann muss eine vollständige Flächen-Integration über sämtliche Grenzflächen des angenommenen Körpers gemacht werden, und dabei lernt sich, dass die Alltags-Vorstellungen sich gegenseitig aufheben.
Es gilt also simpel rho × g × h mit nachstehenden oft nicht explizit genannten Einschränkungen:
  • Keine Kapillarwirkung. Sonst nimmt die Druck auf und weg.
  • Statik: Keine Zu- oder Abflüsse, beschleunigtes System, dynamische Strömung am Messpunkt.
  • Ideales isotropes Fluid, oder beim Fluid in Ruhe halt keine Schubspannungen.
  • Luftdruck an der Wasseroberfläche entweder oben addieren oder überall abziehen. Überhaupt eine freie Oberfläche? Keine Hütchenspielerei.
  • rho konstant über die Tiefe; bei Ozeanen wegen Temperatur und Salinität nicht gegeben und präziser zu erfassen.
  • g konstant über die Tiefe; bei Ozeanen über Kilometer rechnerisch sichtbar aber zu vernachlässigen.
Glas übt keinen Druck aus, im Alltagssinn, und kann als starr angenommen werden (ist aber ein Fluid).
  • Der Behälter mit seinen starren Wänden reagiert auf das Gewicht des Fluides aber mit einer Gegenkraft, und trägt die über ihm liegende Wassersäule. Und nimmt deshalb dem größeren Querschnitt oben diejenigen Gewichts-Anteile ab, die dann als Gewicht nicht mehr nach ganz unten auf eine kleine Bodenfläche wirken, und die deshalb nichts zusätzlich beitragen.
  • Diese einleuchtende Vorstellung wird allerdings kompliziert, wenn der Behälter ein Rohr > Kapillare mit spiralförmigem oder in der Ebene in Schlangenlinien gewundener Mittellinie, und dann keine vertikale Wassersäule direkt drüber, und dann geht nur noch Flächen-Integral.
  • Die größere Bodenfläche als im Hals des Gefäßes trägt auf einer größeren Fläche mehr Gewicht der Säule darüber, aber weil Druck gleich Kraft pro Fläche ist der Druck je Flächeneinheit genauso unabhängig von den Querschnittsflächen an Boden, Hals und dazwischen.
Das psychologische Paradoxon resultiert wohl auf der intuitiven Verwechslung zwischen Gewichtskraft auf die Bodenfläche und dem Druck (pro Flächeneinheit). Die Aussage ist, dass der Druck unabhängig von der Form ist; aber das wäre etwa pro Quadratzentimeter.
  • Wenn Statiker den Sockel für das Gefäß dimensionieren, interessieren sie sich nur für die Gesamtmasse im Behälter plus Behältergewicht. Wenn es um die Wandstärken des Gefäßes nahe Boden geht, ist nur die Höhe ab freier Oberfläche relevant.
  • Bernoulli hat das gleiche Alltagsproblem: 90 % der Laien sind felsenfest überzeugt, dass in bewegter Luft ein höherer Druck vorliegen würde als in ruhender.
Die in der Abschnittsüberschrift genannte Datei:Hydrostatisches Paradoxon3.svg ist momentan auch nicht im Artikel enthalten.
Ahoi --Schiefbauer (Diskussion) 14:12, 15. Mai 2022 (CEST)Beantworten

Für Skeptiker noch ein Beweiß das diese Wp-Seite recht hat ![Quelltext bearbeiten]

Ich kam vom hydrostatischem Gaja-Auftriebskraftwerk und angetrieben Skeptizismus hier auf das Hydrostatisches Paradoxon ! Konnte Anfangs dem Bild nicht so recht glauben und machte folgendes Experiment:

- leere 2 Liter Colaflasche - 1 Colaflaschen-Deckel mit ~5mm Schlauch (eng durchbohrt & dicht durchgewürgt) 1/2m bis 3/4m lang - länge = egal

Füllte das mit Wasser, sofort sah man das hier Wikipedia mit dem Paradoxon recht hatte: Der Wasserstand war GENAU auf gleicher Höhe, egal wie man den Schlauch hielt!

Faszinierend: Dachte ich, es müßte doch eigentlich ein höherer Druck auf den dünnen Druck wirken. @Hubkolbenqerschnitt & sein Druck Gedankenexperiment: 5 Liter Eimer mit einem 1Liter Eimer, mit einem Schlauch zu verbinden, müßte doch 5 mal höherer Druck wirken ? Antwort & Lösung: Pustekuchen ! Richtig: Die Eimer haben 5x unterschiedliches Gewicht, aber wer trägt es denn ? Richtig das Schaaf welches in der Hand hält! Auch die 2Liter Cola-Flasche , hält ein Schaaf in der Hand, ohne zu bemerken, das die 2 Liter Colaflasche, viel mehr als das dünne Schlauchende wiegt. Fazit:z.B Beim Trichter wird der Überstehende schräge Trichterrand in Trichter-Gewicht und nicht in Wassersäulendruck umgewandelt. Warum ? fragen sie mal ihren Höhenmesser xD - Danke xD Späthestens bei einem Lehrer-Experiment mit einem 15-20Liter trichterförmigen Behälter + Schlauch, müßte es selbst der Begriffs-Stutzigste verstehen & sehen, das 1 Schüler das Schlauchende spielend hoch hält, während der Junge mit 20Litern an einem Henkel tragend, anfängt zu stöhnen und zu japsen xD @Wo die Kraft hingeht --79.229.203.219 13:49, 27. Mai 2015 (CEST)Beantworten

die „Erklärung" ist unnötig umständlich[Quelltext bearbeiten]

Zunächst: ich bitte um Vergebung, aber ich halte die Erläuterung für unnötig umständlich.

Wichtig zum Verständnis ist mMn der Umstand, dass Druck in Flüssigkeiten nach allen Seiten wirkt. Prof. Krüger hatte Anfang der 60er Jahre in Tübingen dazu einen wunderbaren Versuch: Er füllte eine Zigarrenkiste randvoll mit Wasser und nagelte sie zu. Dann schoss er mit einem Luftgewehr auf die Kiste. Der Druck der eindringenden, weniger als 1/4 ccm großen "Kugel" drückte die Bretter der Kiste millimeterweit auseinander und erzeugte, verglichen mit der Kleinheit des Geschosses, erstaunlich breite Fugen.

Oder auch: Wenn man eine Zahnpastatube hinten flach zusammendrückt, dann kommt doch Zahnpasta nach vorne heraus.


Nun zur „Erklärung" des hydrostatischen Drucks:

Wären in Bild 1 das Blaue ein Feststoff (ohne die Gefäßwände) und außerdem die roten Striche jeweils die Schalen einer Waage, dann wären die gemessenen Kräfte auf die Waage natürlich nicht gleich.

Da es sich aber um eine Flüssigkeiten handelt wird im rechten Bild der nichtvertikale Anteil des hydrostatischen Drucks auf die Gefäßwände abgeleitet, bzw. bewirkt im linken Bild der Gegendruck der Gefäßwand, dass der hydr. Druck eben so hoch ist, wie beim Bild in der Mitte.


Mir ist klar, dass das KEIN für die Wiki brauchbarer Text ist, aber vielleicht kann ein Spezialist daraus eine Wiki-taugliche Erklärung zaubern?

Ferner: Im ersten Satz der Erklärung muss „Grundfläche" entfernt werden, denn Druck ist Kraft : Fläche. Auch die Worte "in derselben Höhe” sind überflüssig und höchstens verwirrend, da ja vom Druck auf den Gefäßboden gesprochen wird.

Der Satz könnte also lauten: In allen Gefäßen mit demselben Füllstand wirkt derselbe Flüssigkeitsdruck auf den Gefäßboden, unabhängig von der Gefäßgeometrie. Als Konsequenz stellt sich bei kommunizierenden Röhren derselbe (Flüssigkeits-)Pegel ein, unabhängig von der Röhrengeometrie.

Nochmals: ich bitte um Vergebung! Beste Grüße --Curioso~dewiki (Diskussion) 21:09, 18. Jul. 2015 (CEST)Beantworten

Ich finde, da gibt es nichts zu vergeben. Dieses Bildchen mit den Pfeilchen und die dazugehörige "Erklärung" wurde vor einiger Zeit sehr vehement verteidigt. Noch dazu verengt die Zeichnung den hydrostatischen Druck auf die senkrechten Kraftkomponenten, wodurch Information verlorengeht. Ich vergebe also nichts, sondern bedanke mich auch noch bei Dir für Deinen Beitrag, und versuche mich wieder mal darin, den Abschnitt einfach zu löschen. Warum sich eine solche Zeichnung bis in unser Jahrhundert im Gerthsen Physik gehalten hat, ist mir nicht erklärlich. Noch was: Was ist ein Spezialist? Es ist normal, dass sich nicht alle mit dem Gleichen auskennen, wäre ja auch langweilig sonst. --Blauer elephant (Diskussion) 11:33, 2. Dez. 2016 (CET)Beantworten

Scheinbares Paradoxon?[Quelltext bearbeiten]

Wo wir gerade beim Aufräumen sind - was bitte soll ein scheinbares Paradoxon (Einleitung) sein - sowas wie ein weisser Schimmel? Hmm, das kommt nicht hin, ein Paradoxon ist zwar ein scheinbarer Widerspruch, damit wäre ein scheinbarer scheinbarer Widerspruch eine doppelte Verneinung - also ein tatsächlicher Widerspruch. Also ein Paradoxon, das nur so aussieht wie eines, aber keines ist, weil es ja doch ein richtiger Widerspruch ist? Kopfkratzend, --Burkhard (Diskussion) 20:07, 2. Dez. 2016 (CET)Beantworten

Das scheint im Sinne des Zwecks zu sein, daher durchgeführt. --Blauer elephant (Diskussion) 10:48, 5. Dez. 2016 (CET)Beantworten

Diskussionsarchiv?[Quelltext bearbeiten]

Mir scheint, dass die Disk. hier nicht nur scheinbar redundant ist (für identische Themen werden neue Abschnitte erstellt), und plädiere daher dafür, ein tatsächliches Auto-Archiv anzulegen, und hoffe darauf, dass auch dieser Abschnitt bald darin landet. --Blauer elephant (Diskussion) 10:48, 5. Dez. 2016 (CET)Beantworten